Fichier LaTeX : bac2023/bac2023gen_fr_mars_sujet2.tex


Télécharger le fichier original / Zone Membre

% !TeX TXS-program:compile = txs:///pdflatex

\documentclass[french,a4paper,11pt]{article}
\usepackage[margin=2cm,includeheadfoot]{geometry}
\usepackage{ProfLycee}
\RequirePackage[upright]{fourier}
\usepackage{amsmath,amssymb,amstext}
\usepackage[scaled=0.875]{helvet}
\renewcommand\ttdefault{lmtt}
\usepackage{textualicomma}
\usepackage{babel}
\usepackage{esvect}
\usepackage{wrapstuff}
\usepackage{enumitem}
\setlist[enumerate,1]{label=\bfseries\arabic*.}
\setlist[enumerate,2]{label=\bfseries\alph*.}

\newcommand{\session}{2023}
\newcommand{\annee}{2023}
\newcommand{\serie}{Gé.}
\newcommand{\lieu}{Métropole}
\newcommand{\jour}{21}
\newcommand{\mois}{Mars}
\newcommand{\numsujet}{2}
\newcommand{\nomfichier}{[BAC \serie{} \annee{}] \lieu{} (\mois{} \annee)}

\title{\nomfichier}
\usepackage{hyperref}
\usepackage{lastpage}
\usepackage{enumitem}
\usepackage{ibrackets}
\usepackage{fancyhdr}

\hypersetup{pdfauthor={Pierquet},pdftitle={\nomfichier},allbordercolors=white,pdfborder=0 0 0,pdfstartview=FitH}
\lhead{\scriptsize\sffamily BAC \serie{} \session}
\chead{\scriptsize\sffamily \lieu{} - Sujet \numsujet}
\rhead{\scriptsize\sffamily \jour{} \mois{} \annee{}}
\lfoot{}
\cfoot{}
\rfoot{\scriptsize\sffamily - \thepage{}/{}\pageref{LastPage} -}
\setlength{\parindent}{0pt}
%divers
\DeclareMathSymbol{;}\mathbin{operators}{'73}
\newcommand\vect[1]{\vv{#1}}

\begin{document}

\pagestyle{fancy}

{\hfill\Huge \faGraduationCap\hfill~}

\part*{\lieu{}, Bac \serie{}, \jour{} \mois{} \annee, sujet n°\numsujet}

\vspace{0.25cm}

\section*{Exercice 1 \dotfill(5 points)}

\medskip

\emph{Cet exercice est un questionnaire à choix multiple.\\ Pour chaque question, une seule des quatre réponses proposées est exacte. Le candidat 
indiquera sur sa copie le numéro de la question et la réponse choisie. Aucune justification n’est demandée. \\ Aucun point n’est enlevé en l’absence de réponse ou en cas de réponse inexacte.}

\medskip

Un jeu vidéo possède une vaste communauté de joueurs en ligne. Avant de débuter une partie, le joueur doit choisir entre deux « mondes » : soit le monde A, soit le monde B.

\smallskip

On choisit au hasard un individu dans la communauté des joueurs.

Lorsqu'il joue une partie, on admet que :

\begin{itemize}
	\item la probabilité que le joueur choisisse le monde A est égale à $\frac25$ ;
	\item si le joueur choisit le monde A, la probabilité qu’il gagne la partie est de $\frac{7}{10}$ ;
	\item la probabilité que le joueur gagne la partie est de $\frac{12}{25}$.
\end{itemize}

On considère les évènements suivants :

\begin{itemize}
	\item $A$ : « Le joueur choisit le monde A » ;
	\item $B$ : « Le joueur choisit le monde B » ;
	\item $G$ : « Le joueur gagne la partie ».
\end{itemize}

\begin{enumerate}
	\item La probabilité que le joueur choisisse le monde A et gagne la partie est égale à : 
	
	\smallskip
	
	\begin{tblr}{width=\linewidth,colspec={X[l,m]X[l,m]X[l,m]X[l,m]}}
		\textbf{a.}~~$\dfrac{7}{10}$ & \textbf{b.}~~$\dfrac{3}{25}$ & \textbf{c.}~~$\dfrac{7}{25}$ & \textbf{d.}~~$\dfrac{24}{125}$
	\end{tblr}
	\item La probabilité $P_B(G)$ de l’événement $G$ sachant que $B$ est réalisé est égale à :
	
	\smallskip
	
	\begin{tblr}{width=\linewidth,colspec={X[l,m]X[l,m]X[l,m]X[l,m]}}
		\textbf{a.}~~$\dfrac{1}{5}$ & \textbf{b.}~~$\dfrac{1}{3}$ & \textbf{c.}~~$\dfrac{7}{15}$ & \textbf{d.}~~$\dfrac{5}{12}$
	\end{tblr}
\end{enumerate}

Dans la suite de l’exercice, un joueur effectue 10 parties successives. On assimile cette situation à un tirage aléatoire avec remise. On rappelle que la probabilité de gagner une partie est de $\frac{12}{25}$.

\begin{enumerate}
	\item La probabilité, arrondie au millième, que le joueur gagne exactement 6 parties est égale à :
	
	\smallskip
	
	\begin{tblr}{width=\linewidth,colspec={X[l,m]X[l,m]X[l,m]X[l,m]}}
		\textbf{a.}~~$0,859$ & \textbf{b.}~~$0,671$ & \textbf{c.}~~$0,188$ & \textbf{d.}~~$0,187$
	\end{tblr}
	\item On considère un entier naturel $n$ pour lequel la probabilité, arrondie au millième, que le joueur gagne au plus $n$ parties est de $0,207$. Alors :
	
	\smallskip
	
	\begin{tblr}{width=\linewidth,colspec={X[l,m]X[l,m]X[l,m]X[l,m]}}
		\textbf{a.}~~$n=2$ & \textbf{b.}~~$n=3$ & \textbf{c.}~~$n=4$ & \textbf{d.}~~$n=5$
	\end{tblr}
	\item La probabilité que le joueur gagne au moins une partie est égale à :
	
	\smallskip
	
	\begin{tblr}{width=\linewidth,colspec={X[l,m]X[l,m]X[l,m]X[l,m]}}
		\textbf{a.}~~$1-\left(\dfrac{12}{25}\right)^{10}$ & \textbf{b.}~~$\left(\dfrac{13}{25}\right)^{10}$ & \textbf{c.}~~$\left(\dfrac{12}{25}\right)^{10}$ & \textbf{d.}~~$1-\left(\dfrac{13}{25}\right)^{10}$
	\end{tblr}
\end{enumerate}


\vspace{0.5cm}

\section*{Exercice 2 \dotfill(5 points)}

\medskip

Des biologistes étudient l’évolution d’une population d’insectes dans un jardin botanique.

Au début de l’étude la population est de \num{100000} insectes.

Pour préserver l’équilibre du milieu naturel le nombre d’insectes ne doit pas dépasser \num{400000}.

\bigskip

\textbf{Partie A : Étude d’un premier modèle en laboratoire}

\medskip

L’observation de l’évolution de ces populations d’insectes en laboratoire, en l’absence de tout prédateur, montre que le nombre d’insectes augmente de $60$\,\% chaque mois.

En tenant compte de cette observation, les biologistes modélisent l’évolution de la population d’insectes à l’aide d’une suite $\left(u_n\right)$ où, pour tout entier naturel $n$, $u_n$ modélise le nombre d’insectes, exprimé en millions, au bout de $n$ mois. On a donc $u_0 = 0,1$.

\begin{enumerate}
	\item Justifier que pour tout entier naturel $n$ : $u_n = 0,1 \times 1,6^n$.
	\item Déterminer la limite de la suite $\left(u_n\right)$.
	\item En résolvant une inéquation, déterminer le plus petit entier naturel $n$ à partir duquel $u_n > 0,4$.
	\item Selon ce modèle, l’équilibre du milieu naturel serait-il préservé ? Justifier la réponse.
\end{enumerate}

\smallskip

\textbf{Partie B : Étude d’un second modèle}

\medskip

En tenant compte des contraintes du milieu naturel dans lequel évoluent les insectes, les biologistes choisissent une nouvelle modélisation.

Ils modélisent le nombre d’insectes à l’aide de la suite $\left(v_n\right)$, définie par : $v_0=0,1$ et, pour tout 
entier naturel $n$, $v_{n+1}=1,6v_n-1,6v_n^2$, où, pour tout entier naturel $n$, $v_n$ est le nombre d’insectes, exprimé en millions, au bout de $n$ mois.

\begin{enumerate}
	\item Déterminer le nombre d’insectes au bout d’un mois.
	\item On considère la fonction $f$ définie sur l’intervalle $\left[0;\frac12\right]$ par $f(x)=1,6x-1,6x^2$.
	\begin{enumerate}
		\item Résoudre l’équation $f(x)=x$.
		\item Montrer que la fonction $f$ est croissante sur l'intervalle $\left[0;\frac12\right]$.
	\end{enumerate}
	\item 
	\begin{enumerate}
		\item Montrer par récurrence que, pour tout entier naturel $n$, $0 \leqslant v_n \leqslant v_{n+1} \leqslant \frac12$.
		\item Montrer que la suite $\left(v_n\right)$ est convergente.
		
		\smallskip
		
		On note $\ell$ la valeur de sa limite. On admet que $\ell$ est solution de l’équation $f(x)=x$.
		\item Déterminer la valeur de $\ell$. Selon ce modèle, l’équilibre du milieu naturel sera-t-il préservé ? Justifier la réponse.
	\end{enumerate}
	\item On donne ci-dessous la fonction \texttt{seuil}, écrite en langage \textsf{Python}.

\begin{CodePythonLstAlt}*[5.5cm]{center}
def seuil(a) :
	v = 0.1
	n = 0
	while v < a :
		v = 1.6*v - 1.6*v*v
		n = n+1
	return n
\end{CodePythonLstAlt}
	\begin{enumerate}
		\item Qu’observe-t-on si on saisit \texttt{seuil(0.4)} ?
		\item Déterminer la valeur renvoyée par la saisie de \texttt{seuil(0.4)}.
		
		Interpréter cette valeur dans le contexte de l’exercice. 
	\end{enumerate}
\end{enumerate}

\vspace*{5mm}

\section*{Exercice 3 \dotfill(5 points)}

\medskip

Dans l’espace rapporté à un repère orthonormé $\left(O;\vect{\imath},\,\vect{\jmath}, \vect{k} \right)$, on considère : 

\begin{itemize}
	\item le plan $\mathcal{P}_1$ dont une équation cartésienne est $2x + y - z + 2 = 0$,
	\item le plan $\mathcal{P}_2$ passant par le point $B(1;1;2)$ et dont un vecteur normal est $\vv*{n}{2}$ $\begin{pmatrix}1\\-1\\1\end{pmatrix}$.
\end{itemize}

\begin{enumerate}
	\item 
	\begin{enumerate}
		\item Donner les coordonnées d’un vecteur $\vv*{n}{1}$ normal au plan $\mathcal{P}_1$.
		\item On rappelle que deux plans sont perpendiculaires si un vecteur normal à l’un des plans est orthogonal à un vecteur normal à l’autre plan.
		
		Montrer que les plans $\mathcal{P}_1$ et $\mathcal{P}_2$ sont perpendiculaires. 
	\end{enumerate}
	\item 
	\begin{enumerate}
		\item Déterminer une équation cartésienne du plan $\mathcal{P}_2$.
		\item On note $\Delta$ la droite dont une représentation paramétrique est : $\begin{dcases} x=0 \\ y=-2+t \\ z=4 \end{dcases}$, $t \in \mathbb{R}$.
		
		Montrer que la droite $\Delta$ est l’intersection des plans $\mathcal{P}_1$ et $\mathcal{P}_2$. 
	\end{enumerate}
\end{enumerate}

On considère le point $A(1;1;1)$ et on admet que le point $A$ n’appartient ni à  $\mathcal{P}_1$ ni à $\mathcal{P}_2$.

On note $H$ le projeté orthogonal du point $A$ sur la droite $\Delta$.

\begin{enumerate}[resume]
	\item On rappelle que, d’après la question \textbf{2.b}, la droite $\Delta$ est l’ensemble des points $M_t$ de coordonnées $(0;-2+t;t)$ où $t$ désigne un nombre réel quelconque.
	\begin{enumerate}
		\item Montrer que, pour tout réel $t$, $AM_t = \sqrt{2t^2-8t+11}$.
		\item En déduire que $AH = \sqrt{3}$. 
	\end{enumerate}
	\item On note $\mathcal{D}_1$ la droite orthogonale au plan $\mathcal{P}_1$ passant par le point $A$ et $H$ le projeté orthogonal du point $A$ sur le plan $\mathcal{P}_1$.
	\begin{enumerate}
		\item Déterminer une représentation paramétrique de la droite $\mathcal{D}_1$. 
		\item En déduire que le point $H_1$ a pour coordonnées $\left(-\frac13;\frac13;\frac53\right)$.
	\end{enumerate}
	\begin{wrapstuff}[r,abovesep=-1cm]
		\begin{tikzpicture}[x={(-20:5cm)},y={(65:3.33cm)},z={(90:1cm)},line join=bevel]
			\tikzstyle{labelddd} = [inner sep=1.5pt,font=\tiny]
			\coordinate (A) at (0,0,0) ; \coordinate (B) at (1,0,0) ;
			\coordinate (C) at (1,1,0) ; \coordinate (D) at (0,1,0) ;
			\coordinate (V) at ($(D)!0.29!(C)$) ;% \filldraw (V) circle[radius=2pt] ;
			\coordinate (W) at (0.5,1,0) ;% \filldraw (W) circle[radius=2pt] ;
			\coordinate (E) at (0.5,0,-1.5) ; \coordinate (F) at (0.5,0,2.5) ;
			\coordinate (K) at (0.5,{5/7},-3.5) ;
			\coordinate (G) at (0.5,1,2.5) ; \coordinate (H) at (0.5,1,-3.5) ;
			\draw[semithick] (W)--(G)--(F)--(E) (H)--(K) ;
			\draw[semithick] (V)--(D)--(A)--(B)--(C)--(W) ;
			\coordinate (HH) at (0.5,0.29,0) ;% \filldraw (HH) circle[radius=2pt] ;
			\coordinate (HHH) at (0.5,0.57,0) ;% \filldraw (HHH) circle[radius=2pt] ;
			\draw[semithick] (HH) node[labelddd,left] {$H$} --++(0,0,1) node[labelddd,left] {$H_1$} --++(0.28,0,0) node[right,labelddd] {$A$} --++(0,0,-1) node[right,labelddd] {$H_2$}--cycle ;
			\draw[semithick] (0.5,-0.5,0) -- (HH) (HHH)--(0.5,1.85,0) ;
			\draw[semithick,densely dashed] (V)--(W) (W)--(H) (HH)--(HHH) ;
			\draw (0.5,-0.2,0) node[inner sep=1.5pt,font=\footnotesize,left=3pt] {$\Delta$} ;
			\draw (0.025,0,0) node[inner sep=1.5pt,font=\scriptsize,above right] {$\mathcal{P}_2$} ;
			\draw (0.5,1,1.5) node[inner sep=0.5pt,font=\scriptsize,left] {$\mathcal{P}_1$} ;
		\end{tikzpicture}
	\end{wrapstuff}
	\item Soit $H_2$ le projeté orthogonal de $A$ sur le plan $\mathcal{P}_2$.
	
	On admet que $H_2$ a pour coordonnées $\left(\frac43;\frac23;\frac43\right)$ et que $H$ a pour coordonnées $(0;0;2)$.
	
	\medskip
	
	Sur le schéma ci-dessous, les plans $\mathcal{P}_1$ et $\mathcal{P}_2$ sont représentés, ainsi que les points $A$, $H_1$, $H_2$ et $H$.
	
	\medskip
	
	Montrer que $AH_1HH_2$ est un rectangle.
\end{enumerate}

\pagebreak

\section*{Exercice 4 \dotfill(5 points)}

\medskip

On considère la fonction $f$ définie sur $\mathbb{R}$ par $f(x)= = \ln \big(1+\text{e}^{-x}\big)$ , où $\ln$ désigne la fonction logarithme népérien.

\smallskip

On note $\mathcal{C}$ sa courbe représentative dans un repère orthonormé $\left(O;\vect{\imath},\,\vect{\jmath}\right)$.

La courbe $\mathcal{C}$ est tracée ci-dessous. 

\begin{center}
	\begin{tikzpicture}[x=1.5cm,y=1.5cm]
		\draw[semithick] (-3,0)--(3.5,0) (0,-0.5)--(0,3) ;
		\draw (0,0) node[below left=0pt] {0} ;
		\draw[semithick] (1,2pt)--++(0,-4pt) node[below=1pt] {1} ;
		\draw[semithick] (2pt,1)--++(-4pt,0) node[left=1pt] {1} ;
		\draw[black,fill=gray] (0,{ln(2)}) circle[radius=1.5pt] ;
		\draw[thick,red,samples=250,domain=-3:3.5] plot (\x,{ln(1+exp(-\x))}) ;
		\draw (3.25,0) node[above=3pt,red,font=\large] {$\mathcal{C}$} ;
		\draw (1.875,-0.44) node {$T_0$} ;
		\def\AA{2}
		\coordinate (Na) at ({\AA},{ln(1+exp(-\AA)}) ;
		\coordinate (Ma) at ({-\AA},{ln(1+exp(\AA)}) ;
		\coordinate (V) at ($(Ma)!-1!(Na)$) ;
		\coordinate (W) at ($(Na)!-1!(Ma)$) ;
		\clip (-3,-0.5) rectangle (3.5,3) ;
		\draw[semithick,densely dashed,samples=2,domain=-3:3.5] plot (\x,{-0.5*\x+ln(2)}) ;
		\draw[semithick,densely dashed] (V)--(W) ;
		\filldraw[black] (Na) circle[radius=2pt] node[above right] {$N_a$} ;
		\filldraw[black] (Ma) circle[radius=2pt] node[above right] {$M_a$} ;
	\end{tikzpicture}
\end{center}

\begin{enumerate}
	\item 
	\begin{enumerate}
		\item Déterminer la limite de la fonction $f$ en $-\infty$.
		\item Déterminer la limite de la fonction $f$ en $+\infty$. Interpréter graphiquement ce résultat.
		\item On admet que la fonction $f$ est dérivable sur $\mathbb{R}$ et on note $f'$ sa fonction dérivée.
		
		Calculer $f'(x)$ puis montrer que, pour tout nombre réel $x$, $f'(x)=\frac{-1}{1+\text{e}^{x}}$.
		\item Dresser le tableau de variations complet de la fonction $f$ sur $\mathbb{R}$.
	\end{enumerate}
	\item On note $T_0$ la tangente à la courbe $\mathcal{C}$ en son point d’abscisse 0.
	\begin{enumerate}
		\item Déterminer une équation de la tangente $T_0$.
		\item Montrer que la fonction $f$ est convexe sur $\mathbb{R}$.
		\item En déduire que, pour tout nombre réel $x$, on a : $f(x) \geqslant -\frac12 x + \ln(2)$.
	\end{enumerate}
	\item Pour tout nombre réel $a$ différent de 0, on note $M_a$ et $N_a$ les points de la courbe $\mathcal{C}$ d’abscisses respectives $-a$ et $a$. On a donc : $M_a \big(-a;f(-a)\big)$ et $N_a \big(a;f(a)\big)$.
	\begin{enumerate}
		\item Montrer que, pour tout nombre réel $x$, on a : $f(x)-f(-x)=-x$.
		\item En déduire que les droites $T_0$ et $(M_aN_a)$ sont parallèles. 
	\end{enumerate}
\end{enumerate}
\end{document}



On considère le cube $ABCDEFGH$ d’arête 1.

On appelle $I$ le point d’intersection du plan $(GBD)$ avec la droite $(EC)$.

\begin{center}
	\begin{tikzpicture}[x={(0:4cm)},y={(27:2cm)},z={(90:4cm)},line join=bevel]
		\coordinate (A) at (0,0,0) ; \coordinate (B) at (1,0,0) ;
		\coordinate (C) at (1,1,0) ; \coordinate (D) at (0,1,0) ;
		\coordinate (E) at (0,0,1) ; \coordinate (F) at (1,0,1) ;
		\coordinate (G) at (1,1,1) ; \coordinate (H) at (0,1,1) ;
		\coordinate (I) at ({2/3},{2/3},{1/3}) ;
		\coordinate (J) at ($(D)!0.5!(B)$) ;
		\coordinate (U) at ($(E)!-0.16!(C)$) ;%prolongement de (EC) à gauche
		\coordinate (V) at ($(E)!1.19!(C)$) ;%prolongement de (EC) à droite
		\draw[semithick,dashed] (A)--(D)--(C) (D)--(H) ;
		\draw[semithick,densely dashed] (D)--(B) (E)--(C) (D)--(G) ;
		\draw[semithick] (A)--(B)--(F)--(E)--cycle  (B)--(C)--(G)--(F)--cycle (E)--(F)--(G)--(H)--cycle (B)--(G) (U)--(E) (C)--(V) ;
		\foreach \point/\pos in {A/below,B/below,C/below,D/below,E/above,F/above,G/above,H/above,I/below left,J/below left}
			{\filldraw (\point) circle[radius=1.25pt] node[\pos] {$\point$} ;}
	\end{tikzpicture}
\end{center}

L’espace est rapporté au repère $\left(A;\vect{AB},\,\vect{AD},\,\vect{AE}\right)$.

\begin{enumerate}
	\item Donner dans ce repère les coordonnées des points $E$, $C$, $G$.
	\item Déterminer une représentation paramétrique de la droite $(EC)$.
	\item Démontrer que la droite $(EC)$ est orthogonale au plan $(GBD)$.
	\item 
	\begin{enumerate}
		\item Justifier qu’une équation cartésienne du plan $(GBD)$ est : $x + y - z - 1 = 0$.
		\item Montrer que le point $I$ a pour coordonnées $\left(\frac23;\frac23;\frac13\right)$.
		\item En déduire que la distance du point $E$ au plan $(GBD)$ est égale à $\frac{2\sqrt{3}}{3}$.
	\end{enumerate}
	\item 
	\begin{enumerate}
		\item Démontrer que le triangle $BDG$ est équilatéral.
		\item Calculer l’aire du triangle $BDG$. On pourra utiliser le point $J$, milieu du segment $[BD]$.
	\end{enumerate}
	\item Justifier que le volume du tétraèdre $EGBD$ est égal à $\frac13$.
	
	\smallskip
	
	\textit{On rappelle que le volume d’un tétraèdre est donné par : $\mathcal{V}=\frac13 \mathcal{B}h$ où $\mathcal{B}$ est l’aire d’une base du tétraèdre et $h$ est la hauteur relative à cette base.}
\end{enumerate}

\end{document}